Author Topic: EM Drive Developments - related to space flight applications - Thread 2  (Read 3320976 times)

Offline zlspradlin

  • Member
  • Posts: 6
  • USA
  • Liked: 3
  • Likes Given: 2
Thanks for the response, and thank to all of you who even bother reading my posts. I'm so privileged to be here talking with minds like yours. I really do want to keep crack-pot and "pseudoscience" to a minimum, but I'm afraid I've not been classically trained. But I do have a passion & (very) open mind.

As for the Maser, what about this discovery: http://www.nature.com/news/microwave-laser-fulfills-60-years-of-promise-1.11199

Could NASA not handle something like this? I wish I could give NASA the DEA's budget... this is so frustrating.

Quote
The laser light excited the pentacene molecules to an energy level known as a metastable state. Then a microwave passing through the crystal triggered the molecules to relax, releasing a cascade of microwaves of the same wavelength.

It was the same principle as an optical laser. "The signal that came out of it was huge," says Oxborrow, about a hundred million times as powerful as an existing maser. Alone in his lab, "I swore a lot and walked around the corridor about five times talking to myself".

Seems like the goal, one of them anyway, could be to reduce the input energy? Also, presuming this drive is working as it seems that it does, and the fact that we don't understand it very well or at all, this could be one of those "Hobbyist/Hacker Hunches" (which is all I have to contribute at this time, sorry!).

Offline deltaMass

  • Full Member
  • ****
  • Posts: 955
  • A Brit in California
  • Liked: 671
  • Likes Given: 275
I wholeheartedly agree with the intention of the smart people around here, helping the rest of us to filter the science from the chaff.

Nevertheless, I assume there is going to be a break point: either the Emdrive proves to deliver some anomalous thrust or it doesn't.

If it does, such anomaly would maybe go against some of the scientific principles stated here (as very pointedly remarked by frobnicat et al).

But such proof by counterexample would need to be really overwhelming and repeatable in order to be accepted. Is that correct?
You can call me Al  ;D

Offline Rodal

  • Senior Member
  • *****
  • Posts: 5911
  • USA
  • Liked: 6124
  • Likes Given: 5564
...
Really hadn't thought about it, but at large X, isn't f prop X ?  and NT prop X^2/f^3  ??
My recollection (from running numerical examples at the time at which you were planning to run experiments with a Gunn diode) is that at large Xmn, with both m~n simultaneously highest, Xmn increases higher than f, so that 

Xmn (f) ~ f ^y  where y>1

Need y>3/2 in order for thrust force to increase with frequency.

My recollection is that the thrust force predictions using your formula for the Gunn Diode frequency were much higher than the predictions of Shawyer and McCulloch.

EDIT: Also the calculations for Shawyer's Flight Thruster (which I recall was run at higher frequency: 3.85GHz, twice the frequency of NASA's EM Drive tests)

Is my memory correct ?  (too bad that we don't have a good search function to look for things like that  :(  )

Can't remember either, that may have been before I did the units check and noticed the 1/f error (typo on post-it notes) in N (number of photons).

I should go back and set up a graphing program for these formulas.  Cavity dimensions scale as 1/f just to add to the mess.   

I never did get back to the cylindrical w/ dielectric case.   Except for a uniform change, most dielectric surfaces need to include a reflection and you need an iterative program to solve them numerically.  (wrote some of these for optical filter companies in the dim dark past)   It would be nice to have an integral solution that could be optimized.

Just a note:  CoM from spacial symmetry is thought to be local from present cosmology, or so I believe, really haven't checked.

It turns out that, on the mean, all three formulas (Shawyer, McCulloch and Notsosureofit) predict decreasing thrust force with increasing frequency

Notice the subtle effect in Notsosureofit's formula (the only formula that contains the mode shapes) showing that at low frequencies, some of the modes have an increasing force with higher frequency, but this happens over a short range of frequency.

Notsosureofit (not at his office presently) kindly sent me an Excel file, whose image I'm attaching below (i "beautified" it a bit -which is subjective-, and I gave him credit on the image's title, if there are any errors they rest on my shoulders, the credit goes to Notsosureofit)

attachment:  Excel file from Notsosureofit / Image mangled by Rodal  ;)
« Last Edit: 05/11/2015 08:47 pm by Rodal »

Offline Notsosureofit

  • Full Member
  • ****
  • Posts: 691
  • Liked: 747
  • Likes Given: 1729
Thanks for the response, and thank to all of you who even bother reading my posts. I'm so privileged to be here talking with minds like yours. I really do want to keep crack-pot and "pseudoscience" to a minimum, but I'm afraid I've not been classically trained. But I do have a passion & (very) open mind.

As for the Maser, what about this discovery: http://www.nature.com/news/microwave-laser-fulfills-60-years-of-promise-1.11199

Could NASA not handle something like this? I wish I could give NASA the DEA's budget... this is so frustrating.

Quote
The laser light excited the pentacene molecules to an energy level known as a metastable state. Then a microwave passing through the crystal triggered the molecules to relax, releasing a cascade of microwaves of the same wavelength.

It was the same principle as an optical laser. "The signal that came out of it was huge," says Oxborrow, about a hundred million times as powerful as an existing maser. Alone in his lab, "I swore a lot and walked around the corridor about five times talking to myself".

Seems like the goal, one of them anyway, could be to reduce the input energy? Also, presuming this drive is working as it seems that it does, and the fact that we don't understand it very well or at all, this could be one of those "Hobbyist/Hacker Hunches" (which is all I have to contribute at this time, sorry!).

If it works as a maser then it must have a reasonably high index at those frequencies (or it was in some sort of cavity)  Too bad it wasn't frustrum shaped.

OK so it was inside a tuned cavity.

Offline deltaMass

  • Full Member
  • ****
  • Posts: 955
  • A Brit in California
  • Liked: 671
  • Likes Given: 275
Where is the calculation for a photon rocket comparison? When calculating the photon rocket thrust was the input power used, or the input power multiplied by the Q? I don't think I've seen that yet. I seriously need to start keeping a spreadsheet of data, tests, results and discussions. It's too much to keep track of in my head.
Rodal, I appreciate the synopses you've posted in order to answer this, and the comparative work by NotSoSureOfIt also. It seems that, to within an order or so (and there's nothing especially accurate happening with the modelling of results yet), the thrust can be modelled as Q*(photon rocket).

Is this an acceptable handwave? And if it is, it says something important.

Offline zlspradlin

  • Member
  • Posts: 6
  • USA
  • Liked: 3
  • Likes Given: 2
If it works as a maser then it must have a reasonably high index at those frequencies (or it was in some sort of cavity)  Too bad it wasn't frustrum shaped.

OK so it was inside a tuned cavity.

The tuned cavity being a cube, correct? Like the attached image? Would feeding the output of the maser into a frustrum change Shawyer's findings? Would it have an effect on the perceived thrust at all? Would it be better to make the maser in a frustrum rather than feed the output into the resonant cavity? Is there any reason this should not be further investigated in regards to an EM Drive?
« Last Edit: 05/11/2015 09:12 pm by zlspradlin »

Offline frobnicat

  • Full Member
  • ****
  • Posts: 518
  • Liked: 500
  • Likes Given: 151
...

At the "instantaneous scale"  there are collisions between photons and atoms where momentum is transferred and it generates heat but not thrust. When the photons are injected their momentum is p1 and energy is E1. It can only conserve NET momentum if there is a 50/50 probability that momentum is absorbed in each direction, without generating ANY heat at all. As soon as things start getting hot, the probability is not 50/50 anymore, then some of the momentum is not being absorbed as thrust, but rather to heat up the metal. Therefore, the NET momentum in either direction will depend on the difference in the dissipation and attenuation, in each direction.

Todd D.
Yes, the problem is, however, how would this, quantitatively, result in a more efficient (thrust/PowerInput) propellant-less drive than a perfectly-collimated photon rocket.

Another fascinating observation that @frobnicat made early on, is that for photons (whether tunneling, dissipation or another mechanism) to end up producing a more efficient drive than a photon rocket, the photons would have to escape the EM Drive as tachyons, superluminally.

Where is the calculation for a photon rocket comparison? When calculating the photon rocket thrust was the input power used, or the input power multiplied by the Q? I don't think I've seen that yet. I seriously need to start keeping a spreadsheet of data, tests, results and discussions. It's too much to keep track of in my head.

There is, in classical interpretation of SR, no reason to multiply the thrust of a photon rocket by Q as the number of time a quantum of momentum carrier bounced around before leaving the rocket for good and living its own trajectory (by definition : as a real particle) is irrelevant. Only the momentum "lost" by a leaving particle in a direction is a "gained" momentum in the opposite direction for a spacecraft.

For a classical SR interpretation of what is possible for a deep space craft assuming no field/bath/mutable_vacuum to swallow and/or push on, see second half of this fascinating :D post ( 3/ ... )

And the other way around, if a cavity is receiving a flux of particles, there is no way (classically) it can recover more nor less momentum than the initial amount entering the boundary, no matter the path and number and modality of interactions of particles with said cavity, if they are to end their lives within. And not only that would make the emitting system to transfer momentum with no more efficiency than using the equivalent of a photon rocket thrust to blow on a sail  (unless it is sending tachyons), but if the sender of the flux is attached to the receiving cavity then the net momentum delta of the whole process is 0 (assuming nothing leaves).

Only between two different systems exchanging photons back and forth many times a transfer of momentum more efficient (by a factor Q) than the photon rocket limit can occur. This could be used to "beam" a force from system A to system B. But again, if system A and B are rigidly linked within a same craft, this is just a static force within the craft, so the Q factor won't amplify nothing useful. EM internal pressure inside a resonating cavity is proportional to Q but does not help to create net imbalance. And as soon as photons leave (if they leave), there can be net imbalance, but Q is not an amplifying factor of efficiency, there are not more photons leaving (for a given energy input) because they bounced many times before leaving.

I've scratched my head for 15 minutes trying to understand the points made in your previous post about asymmetric dissipation but this is not making sense for me so far, even trying hard to think "out of the box". Will try to address and ask for clarifications when time permits.

edit: blowing on a reflective sail a flux of photon actually allows the emitter to "transmit a force", for a given power, twice that of a photon rocket thrust (best case). Still irrelevant if emitter is rigidly linked to sail : perfectly absorbing sail -> 0 net thrust, perfectly reflecting and collimating sail -> thrust/power=1/c
« Last Edit: 05/11/2015 10:06 pm by frobnicat »

Offline deltaMass

  • Full Member
  • ****
  • Posts: 955
  • A Brit in California
  • Liked: 671
  • Likes Given: 275
@frobnicat:
I totally agree that "Q*(photon rocket thrust)" is a nonsense piece of physics on the face of it, for the same reasons you elaborate. However, I was just pointing out that it simply seems to be a good experimental thrust predictor.

Which, of course, is completely weird if you take the thrust measurements as largely unflawed. And no, I don't know what it points to.

Offline Rodal

  • Senior Member
  • *****
  • Posts: 5911
  • USA
  • Liked: 6124
  • Likes Given: 5564
If it works as a maser then it must have a reasonably high index at those frequencies (or it was in some sort of cavity)  Too bad it wasn't frustrum shaped.

OK so it was inside a tuned cavity.

The tuned cavity being a cube, correct? Like the attached image? Would feeding the output of the maser into a frustrum change Shawyer's findings? Would it have an effect on the perceived thrust at all? Would it be better to make the maser in a frustrum rather than feed the output into the resonant cavity?
All good questions that cannot be answered with the present state of knowledge.  EM Drive researchers have operated with very tiny budgets.

Is there any reason this should not be further investigated in regards to an EM Drive?
Yes, the reason is that NASA Eagleworks doesn't even have the money to test another EM Drive piggybacked in series, the only one they have was made in the living-room of one of the researchers.  A MASER costs hundreds of thousands of dollars, which is way out of their  tiny budget.
« Last Edit: 05/11/2015 10:12 pm by Rodal »

Offline WarpTech

  • Full Member
  • ****
  • Posts: 1407
  • Do it!
  • Statesville, NC
  • Liked: 1453
  • Likes Given: 1925
...

At the "instantaneous scale"  there are collisions between photons and atoms where momentum is transferred and it generates heat but not thrust. When the photons are injected their momentum is p1 and energy is E1. It can only conserve NET momentum if there is a 50/50 probability that momentum is absorbed in each direction, without generating ANY heat at all. As soon as things start getting hot, the probability is not 50/50 anymore, then some of the momentum is not being absorbed as thrust, but rather to heat up the metal. Therefore, the NET momentum in either direction will depend on the difference in the dissipation and attenuation, in each direction.

Todd D.
Yes, the problem is, however, how would this, quantitatively, result in a more efficient (thrust/PowerInput) propellant-less drive than a perfectly-collimated photon rocket.

Another fascinating observation that @frobnicat made early on, is that for photons (whether tunneling, dissipation or another mechanism) to end up producing a more efficient drive than a photon rocket, the photons would have to escape the EM Drive as tachyons, superluminally.

Where is the calculation for a photon rocket comparison? When calculating the photon rocket thrust was the input power used, or the input power multiplied by the Q? I don't think I've seen that yet. I seriously need to start keeping a spreadsheet of data, tests, results and discussions. It's too much to keep track of in my head.

There is, in classical interpretation of SR, no reason to multiply the thrust of a photon rocket by Q as the number of time a quantum of momentum carrier bounced around before leaving the rocket for good and living its own trajectory (by definition : as a real particle) is irrelevant. Only the momentum "lost" by a leaving particle in a direction is a "gained" momentum in the opposite direction for a spacecraft.

For a classical SR interpretation of what is possible for a deep space craft assuming no field/bath/mutable_vacuum to swallow and/or push on, see second half of this fascinating :D post ( 3/ ... )

And the other way around, if a cavity is receiving a flux of particles, there is no way (classically) it can recover more nor less momentum than the initial amount entering the boundary, no matter the path and number and modality of interactions of particles with said cavity, if they are to end their lives within. And not only that would make the emitting system to transfer momentum with no more efficiency than using the equivalent of a photon rocket thrust to blow on a sail  (unless it is sending tachyons), but if the sender of the flux is attached to the receiving cavity then the net momentum delta of the whole process is 0 (assuming nothing leaves).

Only between two different systems exchanging photons back and forth many times a transfer of momentum more efficient (by a factor Q) than the photon rocket limit can occur. This could be used to "beam" a force from system A to system B. But again, if system A and B are rigidly linked within a same craft, this is just a static force within the craft, so the Q factor won't amplify nothing useful. EM internal pressure inside a resonating cavity is proportional to Q but does not help to create net imbalance. And as soon as photons leave (if they leave), there can be net imbalance, but Q is not an amplifying factor of efficiency, there are not more photons leaving (for a given energy input) because they bounced many times before leaving.

I've scratched my head for 15 minutes trying to understand the points made in your previous post about asymmetric dissipation but this is not making sense for me so far, even trying hard to think "out of the box". Will try to address and ask for clarifications when time permits.

edit: blowing on a reflective sail a flux of photon actually allows the emitter to "transmit a force", for a given power, twice that of a photon rocket thrust (best case). Still irrelevant if emitter is rigidly linked to sail : perfectly absorbing sail -> 0 net thrust, perfectly reflecting and collimating sail -> thrust/power=1/c

In steady-state, I would agree with you, but it's not a steady state system. It stores up a lot of photons slowly, then they collectively exert a thrust while depleting the energy stored. Shawyer said that acceleration causes the Q to decrease, it causes the stored energy to decrease. When that is happening, there is more energy and momentum being used for Thrust than there is being input by the source. So just because you have 100W input, does not mean the maximum thrust is based on 100W. The maximum thrust will depend on the total momentum bouncing back and forth at any given time. Accelerating, depletes that energy more quickly than it can be restored by the source.

Frankly, if you had a source of microwaves equal to Q*100W, that photon rocket would probably be more efficient than a frustum at exerting thrust because there are fewer losses.

Todd

« Last Edit: 05/11/2015 10:18 pm by WarpTech »

Offline Rodal

  • Senior Member
  • *****
  • Posts: 5911
  • USA
  • Liked: 6124
  • Likes Given: 5564
...
Frankly, if you had a source of microwaves equal to Q*100W, that photon rocket would probably be more efficient than a frustum at exerting thrust because there are fewer losses.

Todd

Shawyer/PhtnRckt = 2 * Q * designFactor
McCulloch/PhtnRckt = Q * ((cavityLength/smallDiameter) - (cavityLength/bigDiameter))

hence

Shawyer/(Q *PhtnRckt) = 2 * designFactor
McCulloch/( Q * PhtnRckt) =  ((cavityLength/smallDiameter) - (cavityLength/bigDiameter))



The predictive formulas therefore satisfy the "Todd conjecture"

(EmDriveThrustForce/(Q*EmDrivePowerInput) /(PhotonRocketThrust/ (Q*PhotonRocketPowerInput)  < 1

equivalently

(EmDriveThrustForce/(EmDrivePowerInput) /(PhotonRocketThrust/ (PhotonRocketPowerInput)  < 1

if and only if this condition is met:

Shawyer:   designFactor < 0.5
McCulloch:   ((cavityLength/smallDiameter) - (cavityLength/bigDiameter)) < 1


[notice that for smallDiameter approaching zero, as the frustum becomes a cone, the condition is not met]

(At first, rapid glance, the experimental results, seem to satisfy Todd's conjecture,although I have not checked them in detail)


Example:

Shawyer Experimental

Design Factor = 1.23205

does not satisfy condition designFactor< 1/2

but the experimental force is 1/2.5625 of what Shawyer's formula predicts, so the experiments do satisfy it:

(1/2.5625)/(1/2) = 0.78 which is less than 1, hence the experiment satisfies Todd's conjecture

See:

1) http://forum.nasaspaceflight.com/index.php?topic=36313.msg1302455#msg1302455

2) http://forum.nasaspaceflight.com/index.php?topic=29276.msg1276053#msg1276053

________

NOTE: we never got the exact geometry of the experiments from Shawyer (it is not given in his papers), so it was estimated to the best of the abilities of this group
« Last Edit: 05/11/2015 10:57 pm by Rodal »

Offline frobnicat

  • Full Member
  • ****
  • Posts: 518
  • Liked: 500
  • Likes Given: 151
@frobnicat:
I totally agree that "Q*(photon rocket thrust)" is a nonsense piece of physics on the face of it, for the same reasons you elaborate. However, I was just pointing out that it simply seems to be a good experimental thrust predictor.

Which, of course, is completely weird if you take the thrust measurements as largely unflawed. And no, I don't know what it points to.
As you calculated, the internal energy is proportional to Q.  The internal energy is dissipated into heat. So that again points toward a thermal effect.  Vacuum someone says?  Let's look at outgassing from the heated FR4

There seems to be confusion here. No question internal energy is proportional to Q, for a given net power input. But this net power input is not clear (at least to me, still quite clueless when it comes to microwave engineering). There is power going from RF amplifier to cavity, there is power bouncing back to RF amplifier (and maybe some part bouncing forward again ?). What is dissipated where ?

My understanding is that, by definition, stating a net power input (to the cavity) of 50W means that 50W are dissipated in the walls of the cavity. So from this, assuming constant net power input, thermal effects should not be dependant on Q, no ? An effect directly dependant on Q would be EM pressure inside, but unless running Q of a lot more than a million, that is low enough to have negligible impact upon mechanical rigidity of cavity... What other effects would have internal energy (dependant on Q) but not dissipated power (not directly dependant on Q) ?

Or is it to say that, at fixed DC power input to RF amplifier, net power input is so strongly dependant on Q ? There is the bandwidth that changes, the ability to excite within the bandwidth... The dummy load tests at EW are pretty convincing, that on a dummy load the thrust chart is flat. The dummy load bounces no power back to RF amplifier (?). Could it be that higher Q means less net input power to cavity (at fixed DC power input to RF amplifier), more bounced back, and thermal effects are to be investigated in the dissipation of this bounced back power at RF amplifier ?

Sorry, this was my time for random musings. But a clarification on power (net vs forward only) vs internal energy seems on order. Are all the data in N/W talking about net power or just "output of RF amp" ?
« Last Edit: 05/11/2015 10:58 pm by frobnicat »

Offline Rodal

  • Senior Member
  • *****
  • Posts: 5911
  • USA
  • Liked: 6124
  • Likes Given: 5564
...There seems to be confusion here. No question internal energy is proportional to Q, for a given net power input. But this net power input is not clear...
My bad.  I had not thought it through. You are right, Q is actually the contrary of energy losses.   
I deleted the message as it was a non-sequitur.
Thanks for catching the error.  :)
« Last Edit: 05/11/2015 11:18 pm by Rodal »

Offline deltaMass

  • Full Member
  • ****
  • Posts: 955
  • A Brit in California
  • Liked: 671
  • Likes Given: 275
If perfect impedance matching is achieved between amplifier output and cavity input port, then, yes, 50W of amplifier RF power results in 50W being dissipated within the cavity (overwhelmingly as Joule heating of the cavity's conducting walls). Such perfect impedance matching "makes the cavity input port look like" a pure resistor to the amplifier. So that's 50W of heat.

In practise perfect matching is not achieved, but we can get close. The match is quantified by the VSWR and can be directly measured by a VSWR meter. When the match is imperfect, a certain percentage of the forward power is reflected at the cavity port and sent back up the feed.

I don't recall seeing any data on EW's VSWR
« Last Edit: 05/11/2015 11:51 pm by deltaMass »

Offline deltaMass

  • Full Member
  • ****
  • Posts: 955
  • A Brit in California
  • Liked: 671
  • Likes Given: 275
Cartoon break.

Woodward had his Angry Flower.
The Quantum Vacuum has xkcd
http://xkcd.com/1486/

Offline Rodal

  • Senior Member
  • *****
  • Posts: 5911
  • USA
  • Liked: 6124
  • Likes Given: 5564
...
I should go back and set up a graphing program for these formulas.  Cavity dimensions scale as 1/f just to add to the mess.   
...
Great graphing program !

I calculated the natural frequencies (NASA truncated cone Brady et.al cavity) for a cylinder with the Geometrical Mean of the Base Diameters (NOT the natural frequencies of the true truncated cone, which are based on Legendre Associated functions and Spherical Bessel functions.  I calculated the frequencies based on Cylindrical Bessel functions for consistency with your method, without a dielectric). I obtained, up to 5 GHz (mode shapes not shown are cut-off):

{{{"TE", 1, 1, 2}, 1.55384*10^9}, {{"TM", 0, 1, 2},
  1.70454*10^9}, {{"TE", 2, 1, 2}, 1.90601*10^9}, {{"TE", 1, 1, 3},
  2.13611*10^9}, {{"TM", 0, 1, 3}, 2.24811*10^9}, {{"TE", 2, 1, 3},
  2.40446*10^9}, {{"TM", 1, 1, 3}, 2.62296*10^9}, {{"TE", 0, 1, 3},
  2.62296*10^9}, {{"TE", 3, 1, 3}, 2.73659*10^9}, {{"TE", 1, 1, 4},
  2.75153*10^9}, {{"TM", 0, 1, 4}, 2.83936*10^9}, {{"TE", 2, 1, 4},
  2.96469*10^9}, {{"TM", 2, 1, 3}, 3.04616*10^9}, {{"TM", 1, 1, 4},
  3.1445*10^9}, {{"TE", 0, 1, 4}, 3.1445*10^9}, {{"TE", 3, 1, 4},
  3.23989*10^9}, {{"TE", 1, 1, 5}, 3.38206*10^9}, {{"TM", 0, 1, 5},
  3.45389*10^9}, {{"TM", 2, 1, 4}, 3.50529*10^9}, {{"TE", 2, 1, 5},
  3.55764*10^9}, {{"TE", 4, 1, 4}, 3.56052*10^9}, {{"TE", 1, 2, 4},
  3.56478*10^9}, {{"TM", 0, 2, 4}, 3.62321*10^9}, {{"TM", 1, 1, 5},
  3.70881*10^9}, {{"TE", 0, 1, 5}, 3.70881*10^9}, {{"TE", 3, 1, 5},
  3.79002*10^9}, {{"TM", 3, 1, 4}, 3.90228*10^9}, {{"TE", 5, 1, 4},
  3.91419*10^9}, {{"TE", 2, 2, 4}, 4.01286*10^9}, {{"TM", 2, 1, 5},
  4.01926*10^9}, {{"TE", 1, 1, 6}, 4.02059*10^9}, {{"TE", 4, 1, 5},
  4.06751*10^9}, {{"TE", 1, 2, 5}, 4.07124*10^9}, {{"TM", 0, 1, 6},
  4.0812*10^9}, {{"TM", 0, 2, 5}, 4.1225*10^9}, {{"TE", 2, 1, 6},
  4.16937*10^9}, {{"TM", 1, 1, 6}, 4.29908*10^9}, {{"TE", 0, 1, 6},
  4.29908*10^9}, {{"TE", 3, 1, 6}, 4.36934*10^9}, {{"TM", 3, 1, 5},
  4.3698*10^9}, {{"TE", 5, 1, 5}, 4.38044*10^9}, {{"TE", 2, 2, 5},
  4.46883*10^9}, {{"TM", 1, 2, 5}, 4.56527*10^9}, {{"TE", 0, 2, 5},
  4.56527*10^9}, {{"TM", 2, 1, 6}, 4.5696*10^9}, {{"TE", 4, 1, 6},
  4.6121*10^9}, {{"TE", 1, 2, 6}, 4.61539*10^9}, {{"TM", 0, 2, 6},
  4.66067*10^9}, {{"TE", 1, 1, 7}, 4.66384*10^9}, {{"TM", 0, 1, 7},
  4.71619*10^9}, {{"TE", 6, 1, 5}, 4.72106*10^9}, {{"TM", 4, 1, 5},
  4.74952*10^9}, {{"TE", 2, 1, 7}, 4.79269*10^9}, {{"TM", 3, 1, 6},
  4.88078*10^9}, {{"TE", 5, 1, 6}, 4.8903*10^9}, {{"TE", 3, 2, 5},
  4.89129*10^9}, {{"TM", 1, 1, 7}, 4.90595*10^9}, {{"TE", 0, 1, 7},
  4.90595*10^9}, {{"TE", 3, 1, 7}, 4.96763*10^9}, {{"TE", 2, 2, 6},
  4.96963*10^9}}

Perhaps in case you want to calculate the remaining missing m,n modes and populate the full scatter plot to see what it looks like ?

Best site for the Bessel Zeros (to within 15 digits):

http://wwwal.kuicr.kyoto-u.ac.jp/www/accelerator/a4/besselroot.htmlx


Thanks
« Last Edit: 05/12/2015 12:30 am by Rodal »

Offline deltaMass

  • Full Member
  • ****
  • Posts: 955
  • A Brit in California
  • Liked: 671
  • Likes Given: 275
As yet I haven't dipped my toe into the doubtless tasty maths of frustum mode frequencies as a function of frustum dimensions. It would seem best to use the most successful mode from an experimental POV. This would be the so-called TE2012 of EW? or something else? I'm talking about max thrust, which is really the only interesting metric at this stage.

Offline Rodal

  • Senior Member
  • *****
  • Posts: 5911
  • USA
  • Liked: 6124
  • Likes Given: 5564
As yet I haven't dipped my toe into the doubtless tasty maths of frustum mode frequencies as a function of frustum dimensions. It would seem best to use the most successful mode from an experimental POV. This would be the so-called TE2012 of EW? or something else? I'm talking about max thrust, which is really the only interesting metric at this stage.
Just like everything else to do with the EM Drive very tortuous and controversial topic.

I agree that according to the Brady et.al. results the TE012 mode had the highest Thrust/InputPower but they are using TM212 in the vacuum tests because they found TE012 too hard to get reproducible results.

The EM Drive is the tar-baby in Uncle Remus (used in the sense of The Oxford English Dictionary defining the "tar baby" as "a difficult problem which is only aggravated by attempts to solve it")

Quote
The more that Br'er Rabbit fights the Tar-Baby, the more entangled he becomes

And the TE012, TM212 designation is based on the cylindrical shape analogue closest to what the fields look like, as the designation for the truncated cone is not standardized, and the quantum numbers for the truncated cone are different
« Last Edit: 05/12/2015 12:56 am by Rodal »

Offline deltaMass

  • Full Member
  • ****
  • Posts: 955
  • A Brit in California
  • Liked: 671
  • Likes Given: 275
I appreciate that there's no standard nomenclature for the frustum. What about Jang's mode? Shawyer's mode? EW actually gets the smallest N/W value of the three.

Offline LasJayhawk

I'm gonna go deep here.

Thinking about the standing waves, and the way the nodes move back and forth, remind me of something. A millipede on walkabout. Now the millipede can't tell you why he moves, he just knows that he does.

There may be a variable we don't understand in the equations, ( pure freaking magic, PFM) that will balance the force equations out, or maybe it's all measurement error. Who knows. But in any event, the value of thrashing this about cannot be understated.

To those who don't understand, I point you to here: http://www.cnet.com/news/this-is-what-sunset-looks-like-on-mars/#ftag=YHF65cbda0

NASA has given me eyes so that I can see the sunset on another world. Perhaps we can give the next generation the chance to see it in person, and perhaps the star won't be our own.

I'm going to go back to crying now...

Tags:
 

Advertisement NovaTech
Advertisement Northrop Grumman
Advertisement
Advertisement Margaritaville Beach Resort South Padre Island
Advertisement Brady Kenniston
Advertisement NextSpaceflight
Advertisement Nathan Barker Photography
0